Proving the number of commuting pairs of elements in $G$ equals number of conjugacy classes in $G$ times...












3












$begingroup$


The first part of this problem asks to describe $operatorname{Hom}(mathbb{Z}^2,G)$ as a subset of $G times G$ which turned out that $operatorname{Hom}(mathbb{Z}^2,G)$ is the set of pairs of elements which commute in G.



Now, I am trying to understand, and later show the fact that



$#operatorname{Hom}(mathbb{Z}^2,G) = #G cdot N$



where $N$ is the number of orbits of the group action of conjugation by $G$ on itself.



My first guess was to try constructing a bijection between the two sides of the equality but I am having a hard time understanding $#G cdot N$ as a set. Is this a correct approach?










share|cite|improve this question











$endgroup$

















    3












    $begingroup$


    The first part of this problem asks to describe $operatorname{Hom}(mathbb{Z}^2,G)$ as a subset of $G times G$ which turned out that $operatorname{Hom}(mathbb{Z}^2,G)$ is the set of pairs of elements which commute in G.



    Now, I am trying to understand, and later show the fact that



    $#operatorname{Hom}(mathbb{Z}^2,G) = #G cdot N$



    where $N$ is the number of orbits of the group action of conjugation by $G$ on itself.



    My first guess was to try constructing a bijection between the two sides of the equality but I am having a hard time understanding $#G cdot N$ as a set. Is this a correct approach?










    share|cite|improve this question











    $endgroup$















      3












      3








      3


      1



      $begingroup$


      The first part of this problem asks to describe $operatorname{Hom}(mathbb{Z}^2,G)$ as a subset of $G times G$ which turned out that $operatorname{Hom}(mathbb{Z}^2,G)$ is the set of pairs of elements which commute in G.



      Now, I am trying to understand, and later show the fact that



      $#operatorname{Hom}(mathbb{Z}^2,G) = #G cdot N$



      where $N$ is the number of orbits of the group action of conjugation by $G$ on itself.



      My first guess was to try constructing a bijection between the two sides of the equality but I am having a hard time understanding $#G cdot N$ as a set. Is this a correct approach?










      share|cite|improve this question











      $endgroup$




      The first part of this problem asks to describe $operatorname{Hom}(mathbb{Z}^2,G)$ as a subset of $G times G$ which turned out that $operatorname{Hom}(mathbb{Z}^2,G)$ is the set of pairs of elements which commute in G.



      Now, I am trying to understand, and later show the fact that



      $#operatorname{Hom}(mathbb{Z}^2,G) = #G cdot N$



      where $N$ is the number of orbits of the group action of conjugation by $G$ on itself.



      My first guess was to try constructing a bijection between the two sides of the equality but I am having a hard time understanding $#G cdot N$ as a set. Is this a correct approach?







      abstract-algebra group-theory group-actions






      share|cite|improve this question















      share|cite|improve this question













      share|cite|improve this question




      share|cite|improve this question








      edited Dec 3 '18 at 1:44









      the_fox

      2,58711533




      2,58711533










      asked Dec 3 '18 at 0:58









      Richard VillalobosRichard Villalobos

      1567




      1567






















          1 Answer
          1






          active

          oldest

          votes


















          3












          $begingroup$

          Write $text{Cl}(a)$ for the conjugacy class of $ain G$, and $C(a)$ for the centralizer of $a$ in $G$.



          Note that $(a,b)$ is a commuting pair if and only if $bin C(a)$. Thus the number of commuting pairs is (counting by the first element in the pair)
          $$text{# pairs} = sum_{ain G} |C(a)|.$$



          Now, choose representatives $a_1,dotsc, a_N$ of the $N$ conjugacy classes. If $g,hin G$ are conjugate, then $|C(g)| = |C(h)|$, so we may rewrite the equation above as
          $$text{# pairs} = sum_{i=1}^N |text{Cl}(a_i)|cdot |C(a_i)|.$$
          By the orbit-stabilizer theorem, $|text{Cl}(a_i)| = |G : C(a_i)|$, so the foregoing simplifies to
          $$text{# pairs} = sum_{i=1}^n |G:C(a_i)|cdot |C(a_i)| = sum_{i=1}^N |G| = text{#}Gcdot N.$$






          share|cite|improve this answer











          $endgroup$













            Your Answer





            StackExchange.ifUsing("editor", function () {
            return StackExchange.using("mathjaxEditing", function () {
            StackExchange.MarkdownEditor.creationCallbacks.add(function (editor, postfix) {
            StackExchange.mathjaxEditing.prepareWmdForMathJax(editor, postfix, [["$", "$"], ["\\(","\\)"]]);
            });
            });
            }, "mathjax-editing");

            StackExchange.ready(function() {
            var channelOptions = {
            tags: "".split(" "),
            id: "69"
            };
            initTagRenderer("".split(" "), "".split(" "), channelOptions);

            StackExchange.using("externalEditor", function() {
            // Have to fire editor after snippets, if snippets enabled
            if (StackExchange.settings.snippets.snippetsEnabled) {
            StackExchange.using("snippets", function() {
            createEditor();
            });
            }
            else {
            createEditor();
            }
            });

            function createEditor() {
            StackExchange.prepareEditor({
            heartbeatType: 'answer',
            autoActivateHeartbeat: false,
            convertImagesToLinks: true,
            noModals: true,
            showLowRepImageUploadWarning: true,
            reputationToPostImages: 10,
            bindNavPrevention: true,
            postfix: "",
            imageUploader: {
            brandingHtml: "Powered by u003ca class="icon-imgur-white" href="https://imgur.com/"u003eu003c/au003e",
            contentPolicyHtml: "User contributions licensed under u003ca href="https://creativecommons.org/licenses/by-sa/3.0/"u003ecc by-sa 3.0 with attribution requiredu003c/au003e u003ca href="https://stackoverflow.com/legal/content-policy"u003e(content policy)u003c/au003e",
            allowUrls: true
            },
            noCode: true, onDemand: true,
            discardSelector: ".discard-answer"
            ,immediatelyShowMarkdownHelp:true
            });


            }
            });














            draft saved

            draft discarded


















            StackExchange.ready(
            function () {
            StackExchange.openid.initPostLogin('.new-post-login', 'https%3a%2f%2fmath.stackexchange.com%2fquestions%2f3023455%2fproving-the-number-of-commuting-pairs-of-elements-in-g-equals-number-of-conjug%23new-answer', 'question_page');
            }
            );

            Post as a guest















            Required, but never shown

























            1 Answer
            1






            active

            oldest

            votes








            1 Answer
            1






            active

            oldest

            votes









            active

            oldest

            votes






            active

            oldest

            votes









            3












            $begingroup$

            Write $text{Cl}(a)$ for the conjugacy class of $ain G$, and $C(a)$ for the centralizer of $a$ in $G$.



            Note that $(a,b)$ is a commuting pair if and only if $bin C(a)$. Thus the number of commuting pairs is (counting by the first element in the pair)
            $$text{# pairs} = sum_{ain G} |C(a)|.$$



            Now, choose representatives $a_1,dotsc, a_N$ of the $N$ conjugacy classes. If $g,hin G$ are conjugate, then $|C(g)| = |C(h)|$, so we may rewrite the equation above as
            $$text{# pairs} = sum_{i=1}^N |text{Cl}(a_i)|cdot |C(a_i)|.$$
            By the orbit-stabilizer theorem, $|text{Cl}(a_i)| = |G : C(a_i)|$, so the foregoing simplifies to
            $$text{# pairs} = sum_{i=1}^n |G:C(a_i)|cdot |C(a_i)| = sum_{i=1}^N |G| = text{#}Gcdot N.$$






            share|cite|improve this answer











            $endgroup$


















              3












              $begingroup$

              Write $text{Cl}(a)$ for the conjugacy class of $ain G$, and $C(a)$ for the centralizer of $a$ in $G$.



              Note that $(a,b)$ is a commuting pair if and only if $bin C(a)$. Thus the number of commuting pairs is (counting by the first element in the pair)
              $$text{# pairs} = sum_{ain G} |C(a)|.$$



              Now, choose representatives $a_1,dotsc, a_N$ of the $N$ conjugacy classes. If $g,hin G$ are conjugate, then $|C(g)| = |C(h)|$, so we may rewrite the equation above as
              $$text{# pairs} = sum_{i=1}^N |text{Cl}(a_i)|cdot |C(a_i)|.$$
              By the orbit-stabilizer theorem, $|text{Cl}(a_i)| = |G : C(a_i)|$, so the foregoing simplifies to
              $$text{# pairs} = sum_{i=1}^n |G:C(a_i)|cdot |C(a_i)| = sum_{i=1}^N |G| = text{#}Gcdot N.$$






              share|cite|improve this answer











              $endgroup$
















                3












                3








                3





                $begingroup$

                Write $text{Cl}(a)$ for the conjugacy class of $ain G$, and $C(a)$ for the centralizer of $a$ in $G$.



                Note that $(a,b)$ is a commuting pair if and only if $bin C(a)$. Thus the number of commuting pairs is (counting by the first element in the pair)
                $$text{# pairs} = sum_{ain G} |C(a)|.$$



                Now, choose representatives $a_1,dotsc, a_N$ of the $N$ conjugacy classes. If $g,hin G$ are conjugate, then $|C(g)| = |C(h)|$, so we may rewrite the equation above as
                $$text{# pairs} = sum_{i=1}^N |text{Cl}(a_i)|cdot |C(a_i)|.$$
                By the orbit-stabilizer theorem, $|text{Cl}(a_i)| = |G : C(a_i)|$, so the foregoing simplifies to
                $$text{# pairs} = sum_{i=1}^n |G:C(a_i)|cdot |C(a_i)| = sum_{i=1}^N |G| = text{#}Gcdot N.$$






                share|cite|improve this answer











                $endgroup$



                Write $text{Cl}(a)$ for the conjugacy class of $ain G$, and $C(a)$ for the centralizer of $a$ in $G$.



                Note that $(a,b)$ is a commuting pair if and only if $bin C(a)$. Thus the number of commuting pairs is (counting by the first element in the pair)
                $$text{# pairs} = sum_{ain G} |C(a)|.$$



                Now, choose representatives $a_1,dotsc, a_N$ of the $N$ conjugacy classes. If $g,hin G$ are conjugate, then $|C(g)| = |C(h)|$, so we may rewrite the equation above as
                $$text{# pairs} = sum_{i=1}^N |text{Cl}(a_i)|cdot |C(a_i)|.$$
                By the orbit-stabilizer theorem, $|text{Cl}(a_i)| = |G : C(a_i)|$, so the foregoing simplifies to
                $$text{# pairs} = sum_{i=1}^n |G:C(a_i)|cdot |C(a_i)| = sum_{i=1}^N |G| = text{#}Gcdot N.$$







                share|cite|improve this answer














                share|cite|improve this answer



                share|cite|improve this answer








                edited Dec 3 '18 at 2:04

























                answered Dec 3 '18 at 1:58









                rogerlrogerl

                17.4k22746




                17.4k22746






























                    draft saved

                    draft discarded




















































                    Thanks for contributing an answer to Mathematics Stack Exchange!


                    • Please be sure to answer the question. Provide details and share your research!

                    But avoid



                    • Asking for help, clarification, or responding to other answers.

                    • Making statements based on opinion; back them up with references or personal experience.


                    Use MathJax to format equations. MathJax reference.


                    To learn more, see our tips on writing great answers.




                    draft saved


                    draft discarded














                    StackExchange.ready(
                    function () {
                    StackExchange.openid.initPostLogin('.new-post-login', 'https%3a%2f%2fmath.stackexchange.com%2fquestions%2f3023455%2fproving-the-number-of-commuting-pairs-of-elements-in-g-equals-number-of-conjug%23new-answer', 'question_page');
                    }
                    );

                    Post as a guest















                    Required, but never shown





















































                    Required, but never shown














                    Required, but never shown












                    Required, but never shown







                    Required, but never shown

































                    Required, but never shown














                    Required, but never shown












                    Required, but never shown







                    Required, but never shown







                    Popular posts from this blog

                    Quarter-circle Tiles

                    build a pushdown automaton that recognizes the reverse language of a given pushdown automaton?

                    Mont Emei